Soluzioni dei test di ammissione

Scuola Normale Superiore, Sant'Anna, Indam, etc. Cosa studiare, come prepararsi.
Avatar utente
Gauss_87
Messaggi: 294
Iscritto il: 21 gen 2006, 17:20
Località: Pisa

Messaggio da Gauss_87 »

aursic ha scritto: - dimostrare (o tentare di farlo) che l'unica coppia (ordinata, per essere pignolo) di numeri naturali
$ (a,b) $ tali che $ a^b=b^a $ è $ (2,4) $;
Ordinata nel senso che $ a<b $ ???
Altrimenti: $ 1^1 = 1^1 $ o no ???
Considerate la vostra semenza: fatte non foste a viver come bruti, ma per seguir virtute e canoscenza
Avatar utente
Gauss_87
Messaggi: 294
Iscritto il: 21 gen 2006, 17:20
Località: Pisa

Messaggio da Gauss_87 »

Che ne dite di questa (presunta) dimostrazione su quell'unica coppia ordinata?
L'ho scritta di getto, è la prima che mi è venuta in mente, quindi non vi assicuro niente.

Intanto posso eliminare i casi banali in cui $ a=0 $ e $ a=1 $ perchè danno soluzioni impossibili, quindi considero $ 2 \leq a < b $

---------------------------------

EDIT: dato che il Tex dà problemi provo a scrivere la soluzione in 2 post
Considerate la vostra semenza: fatte non foste a viver come bruti, ma per seguir virtute e canoscenza
Avatar utente
Gauss_87
Messaggi: 294
Iscritto il: 21 gen 2006, 17:20
Località: Pisa

Messaggio da Gauss_87 »

, pertanto $ b \geq 3 \Rightarrow a^b = b^a \geq 9 $.

$ a^b = b^a \Leftrightarrow \log_{a}a^b = \log_{a}b^a \Leftrightarrow b = a \log_{a}b $ (1).

Dalla (1), siccome $ b>a $, ricavo che $ b $ è una potenza di $ a $, quindi $ b = a^x $ per $ x \in Z^+ - \{1\} $.

La (1) diventa $ a^x = a \cdot x \Leftrightarrow a^{x-1} = x \Leftrightarrow a = \sqrt[x-1]{x} $ e per $ x = 2 \Rightarrow a = 2 $, $ b=4 $ come da tesi.

Adesso devo dimostrare che $ (2,4) $ è unica,

cioè basta dimostrare $ 1 < \sqrt[x-1]{x} \leq 2 $.

La prima disuguaglianza è banalmente vera dal fatto che $ x \in Z^+ - \{1\} $.

$ \sqrt[x-1]{x} \leq 2 \Leftrightarrow 2x \leq 2^x $ per induzione:

$ x = 2 $ proprietà vera.

$ 2x \leq 2^x \Rightarrow 2(x+1) \leq 2^{x+1} $:

Dall'ipotesi induttiva $ 2x \leq 2^x \Leftrightarrow 2(x+1) \leq 2^x + 2 \underbrace{\leq}_{spero} 2^{x+1} $

e con un paio di conti la speranza è verificata $ \forall x \in Z^+ - \{1\} $, da cui la tesi.
Considerate la vostra semenza: fatte non foste a viver come bruti, ma per seguir virtute e canoscenza
__Cu_Jo__
Messaggi: 207
Iscritto il: 10 mar 2005, 07:39

Messaggio da __Cu_Jo__ »

HomoPatavinus mi pare che il libro che cerchi tu(con le soluzioni delle prove di matematica della normale di pisa) sia fuori catalogo.Sono andato addirittura al deposito della zanichelli, e non ce l'avevano.

Gauss me lo spieghi come fai a dire che b deve essere una potenza di a?
Avatar utente
Gauss_87
Messaggi: 294
Iscritto il: 21 gen 2006, 17:20
Località: Pisa

Messaggio da Gauss_87 »

__Cu_Jo__ ha scritto: Gauss me lo spieghi come fai a dire che b deve essere una potenza di a?
Si certo, guarda la (1):

$ b = a \cdot \log_a b \in Z^+ $ (2)

con $ b > a $ quindi

$ \log_a b > 1 $, $ a \in Z^+ \Rightarrow \log_a b \in Z^+ $ affinchè sia soddisfatta l'appartenza agli interi positivi nella (2).

Infine $ \log_a b \in Z^+ $ significa che b è una potenza di a!!!

Credo di aver fatto bene, no ???
Considerate la vostra semenza: fatte non foste a viver come bruti, ma per seguir virtute e canoscenza
Avatar utente
Paoloca
Messaggi: 88
Iscritto il: 18 apr 2005, 18:11

Re: Soluzioni dei test di ammissione

Messaggio da Paoloca »

HomoPatavinus ha scritto:esiste un sito in cui è possibile trovare tutte le soluzioni ( oppure una parte) dei passati test di ammissione alla normale ?
Ecco il 2002:


http://dida.sns.it/dida2/cl/02-03/Ammis ... 0I%20anno/
__Cu_Jo__
Messaggi: 207
Iscritto il: 10 mar 2005, 07:39

Messaggio da __Cu_Jo__ »

Gauss_87 ha scritto:
Si certo, guarda la (1):

$ b = a \cdot \log_a b \in Z^+ $ (2)

con $ b > a $ quindi

$ \log_a b > 1 $, $ a \in Z^+ \Rightarrow \log_a b \in Z^+ $ affinchè sia soddisfatta l'appartenza agli interi positivi nella (2).

Infine $ \log_a b \in Z^+ $ significa che b è una potenza di a!!!

Credo di aver fatto bene, no ???
Guarda...forse sono completamente fuso(l'esame di maturità gioca brutti scherzi...),però così dimostri solo che b è un multiplo di a.O no?Corregimi se sbaglio.
darkcrystal
Messaggi: 706
Iscritto il: 14 set 2005, 11:39
Località: Chiavari

Messaggio da darkcrystal »

Forse sono fuso anch'io, ma mi sembra che Gauss_87 abbia ragione: infatti $ log_a b $ è intero, e quindi b è a elevato ad un esponente intero e strettamente maggiore di 1, e pertanto è potenza di a.

Sperando di non dire sciocchezza, ciao a tutti!
"Solo due cose sono infinite: l'universo e la stupidità dell'uomo, e non sono tanto sicuro della prima" - Einstein

Membro dell'EATO
__Cu_Jo__
Messaggi: 207
Iscritto il: 10 mar 2005, 07:39

Messaggio da __Cu_Jo__ »

Perfetto, ho capito.
Avatar utente
Gauss_87
Messaggi: 294
Iscritto il: 21 gen 2006, 17:20
Località: Pisa

Messaggio da Gauss_87 »

__Cu_Jo__ ha scritto: Guarda...forse sono completamente fuso(l'esame di maturità gioca brutti scherzi...),però così dimostri solo che b è un multiplo di a.O no?Corregimi se sbaglio.
Direi che sbagli...
Considerate la vostra semenza: fatte non foste a viver come bruti, ma per seguir virtute e canoscenza
Avatar utente
Ani-sama
Messaggi: 418
Iscritto il: 19 feb 2006, 21:38
Località: Piacenza
Contatta:

Messaggio da Ani-sama »

aursic ha scritto:- dimostrare (o tentare di farlo) che l'unica coppia (ordinata, per essere pignolo) di numeri naturali
$ (a,b) $ tali che $ a^b=b^a $ è $ (2,4) $;
Ci provo anch'io...

Dunque:


Teorema

L'unica coppia di numeri naturali $ $(a,b)$ $ tali che $ $a^b=b^a$ $ è $ $(2,4)$ $.

Supponiamo, per cominciare, che $ $b>a$ $. Poniamo dunque $ $a=n$ $ e $ $b=n+k$ $ per un qualche $ $k \in \mathbb{N}$ $. È facile verificare a mano i casi piccoli, ossia tutte le coppie $ $(n,k)$ $ che si possono formare ponendo $ $n \leq 2, k \leq 2$ $. Si nota che con $ $n=2, k=2$ $ si ha l'uguaglianza cercata. Ora, per dimostrare che questa è unica, dimostriamo la seguente:

$ $n^{n+k} > {(n+k)}^n$ $, con $ $n > 2, \forall k$ $

Dividiamo ambo i membri per $ $n^n$ $, ottenendo:

$ $n^k > {\left(1 + \frac{k}{n}\right)}^n$ $

Ora estriamo la radice $ $k\textrm{-esima}$ $:

$ $n > {\left(1 + \frac{k}{n}\right)}^{\frac{n}{k}}$ $

Scriviamo l'espressione in questo modo:

$ $n > {\left(1 + \frac{1}{\frac{n}{k}}\right)}^{\frac{n}{k}}$ $

Ma per $ $n >2$ $ tale espressione è sicuramente verificata, dal momento che, per il noto limite:

$ $\lim_{n \rightarrow +\infty} {\left(1 + \frac{1}{\frac{n}{k}}\right)}^{\frac{n}{k}}=e <3$ $

La tesi risulta dunque provata. Si procede del tutto analogamente (anzi, simmetricamente) considerando $ $b < a$ $...
...
Avatar utente
Gauss_87
Messaggi: 294
Iscritto il: 21 gen 2006, 17:20
Località: Pisa

Messaggio da Gauss_87 »

Ani-sama ha scritto:
aursic ha scritto:- dimostrare (o tentare di farlo) che l'unica coppia (ordinata, per essere pignolo) di numeri naturali
$ (a,b) $ tali che $ a^b=b^a $ è $ (2,4) $;
Scriviamo l'espressione in questo modo:

$ $n > {\left(1 + \frac{1}{\frac{n}{k}}\right)}^{\frac{n}{k}}$ $

Ma per $ $n >2$ $ tale espressione è sicuramente verificata, dal momento che, per il noto limite:

$ $\lim_{n \rightarrow +\infty} {\left(1 + \frac{1}{\frac{n}{k}}\right)}^{\frac{n}{k}}=e <3$ $

La tesi risulta dunque provata. Si procede del tutto analogamente (anzi, simmetricamente) considerando $ $b < a$ $...
Direi che la dimostrazione è carina anche se ad essere pignoli dovresti dimostrare che

$ {\left(1 + \frac{1}{\frac{n}{k}}\right)}^{\frac{n}{k}}$ $ è crescente anche se è un risultato noto! :lol:
Complimenti ad Ani-sama !!!
Considerate la vostra semenza: fatte non foste a viver come bruti, ma per seguir virtute e canoscenza
__Cu_Jo__
Messaggi: 207
Iscritto il: 10 mar 2005, 07:39

Messaggio da __Cu_Jo__ »

Ragazzi,a questo punto ci provo anche io :D!Possiamo scrivere la disuguaglianza in questo modo:
$ \displaystyle a^{\frac{1}{a}} = b^{\frac{1}{b}} $
La funzione $ \displaystyle y = x^{\frac{1}{x}} $ è strettamente crescente per $ 0 < x \le e $ e strettamente descrescente per $ x \ge e $.Basta fare la derivata prima:
$ \displaystyle \ln y = \frac{1}{x}\ln x \Rightarrow \frac{1}{y}y' = \frac{{1 - \ln x}}{{x^2 }} \Rightarrow y' = x^{\frac{1}{x}} \frac{{1 - \ln x}}{{x^2 }} $

Se $ a<b $ allora a può assumere solo i valori 1 e 2. Infatti a e b non si possono trovare entrambi negli intervalli $ \left] {0,e} \right],\left[ {e, + \infty } \right[ $. Per $ a=1 $ abbiamo $ b=1 $ che non possiamo accettare perchè $ a<b $. Se $ a=2 $ bisogna risolvere l'equazione $ 2^b=b^2 $.Questa si può risolvere a tentativi tenendo conto che $ 2^b $ cresce più rapidamente rispetto a $ b^2 $(a b=5 ci possiamo già fermare).
darkcrystal
Messaggi: 706
Iscritto il: 14 set 2005, 11:39
Località: Chiavari

Messaggio da darkcrystal »

Una piccola nota: da $ 2^b=b^2 $ si nota che b è una potenza di 2, pertanto gli unici tentativi da fare sono (1,2,4).
Ciao!
"Solo due cose sono infinite: l'universo e la stupidità dell'uomo, e non sono tanto sicuro della prima" - Einstein

Membro dell'EATO
Avatar utente
Gauss_87
Messaggi: 294
Iscritto il: 21 gen 2006, 17:20
Località: Pisa

Messaggio da Gauss_87 »

__Cu_Jo__ ha scritto:Ragazzi,a questo punto ci provo anche io :D!Possiamo scrivere la disuguaglianza in questo modo:
$ \displaystyle a^{\frac{1}{a}} = b^{\frac{1}{b}} $
Direi che la dimostrazione è molto breve quindi più interessante della mia, a parte il fatto che alla Normale ti avrebbero mandato a casa se chiami quella lì disuguaglianza !!! :lol: !!! :lol: è con l' = !!! :lol:
Si fa per skerzare !!! Bye
Considerate la vostra semenza: fatte non foste a viver come bruti, ma per seguir virtute e canoscenza
Rispondi